Saturday 11 October 2014

quantum field theory - $phi^{4}$ Propagator - Feynman Diagram: internal vertex that loops back to itself


In all that follows I'll be dealing with everything massless.


The free, massless propagator ($\mathcal{L} = \int d^{4}x \left(\partial \phi(x) \right)^{2} $) is supposedly given by $G_{0}(x,y) = c (x-y)^{-2}$, where I believe $c = \frac{1}{4\pi^{2}}$.



I'm trying to calculate the propagator in $\phi^{4}$-theory, specifically the contribution due to this diagram: enter image description here


Using the Feynman rules in position space, I believe that I should be getting a contribution of the form: $$ C(x_{1},x_{2}) = -i\lambda \int d^{4}u\ G_{0}(x_{1}, u) G_{0}(u,u) G_{0}(u,x_{2}) $$


However, here is my question: why do I get $G_{0}(u,u) = c (u-u)^{-2} = \mathrm{undefined}$? There's no way I can see to evaluate this integral.


How do I deal with this? Maybe I've got the wrong order of variables? I'm new to these kinds of calculations.




No comments:

Post a Comment

Understanding Stagnation point in pitot fluid

What is stagnation point in fluid mechanics. At the open end of the pitot tube the velocity of the fluid becomes zero.But that should result...